Precalculus (6th Edition) Blitzer

Published by Pearson
ISBN 10: 0-13446-914-3
ISBN 13: 978-0-13446-914-0

Chapter 2 - Test - Page 434: 25

Answer

Domain: $(-\infty,\infty)$ See graph.

Work Step by Step

Step 1. The domain requirement is that the denominator can not be zero, which gives $x^2+3\ne0$. Thus the domain can be written as: $(-\infty,\infty)$ Step 2. As $f(-x)=f(x)$, the function is even, and we can not identify any vertical asymptotes. Step 3. As $x\to\pm\infty, y\to4$, we find a horizontal asymptote as $y=4$ Step 4. See graph.
Update this answer!

You can help us out by revising, improving and updating this answer.

Update this answer

After you claim an answer you’ll have 24 hours to send in a draft. An editor will review the submission and either publish your submission or provide feedback.